1. Applicants to our graduate program have GRE Quantitative Reasoning scores that can be modelled by a Normal random variable with a mean of 155 and a standard deviation of 12. a. What is the probabil

Answers

Answer 1

The probability of getting GRE Quantitative Reasoning scores greater than 170 is 0.8944.

Normal random variable with a mean of 155. The given GRE Quantitative Reasoning scores can be modeled as a Normal random variable. The mean of the given Normal distribution is 155 and its standard deviation is 12. GRE Quantitative Reasoning scores for some different parts as given below. Part a: Probability of getting GRE Quantitative Reasoning scores greater than 170 Z =

(X - μ) / σZ

= (170 - 155) / 12

Z = 1.25

Probability of getting GRE Quantitative Reasoning scores greater than 170 is 0.8944.

To know more about Quantitative visit :-

https://brainly.com/question/32236127

#SPJ11


Related Questions

Suppose X and Y are independent, identically distributed random variables that are uniform on the interval [0, 20], where 0 > 0. (a) (10 pts). Show that the distribution of X/0 is independent of 0. (b) (20 pts). Without computing the distribution of X/Y, find E(X/Y) and Var(X/Y). (c) (10 pts). For k>0 and 1>0, compute E(0-1X/Yk). (d) (30 pts). Find the density function of Z = X/Y. (e) (30 pts). Suppose that X₁, X2, same distribution as X. Let X(n) X, are independent with the max(X₁, X2, ..., X). Find an expression for c so that X(n)/c is a lower 100(1-a)% confidence bound for 0, that is e satisfies Pr(0> X(n)/c) 1-a

Answers

a) Show that the distribution of X/0 is independent of 0.In the Uniform [0, 20] distribution, the probability density function (pdf) is constant between 0 and 20. For example, for any a, b such that 0 ≤ a ≤ b ≤ 20:P(a ≤ X ≤ b) = (b − a)/20For 0 > 0, we have to multiply this pdf by 1/0 for any x > 0 and 0 otherwise. We have:When x = 0, this expression evaluates to 0/0, so we use L'Hopital's rule:lim(1/x) = 0 as x → 0, so we obtain:P(X/0 ≤ t) = P(X ≤ 0) = 0for any t > 0. Thus, the distribution of X/0 is degenerate at 0, and is independent of 0.b) Without computing the distribution of X/Y, find E(X/Y) and Var(X/Y).

The expected value of X/Y is E(X/Y) = E(X)E(1/Y)As X and Y are independent and identically distributed uniform [0, 20] variables, we have E(X) = 10 and: E(1/Y) = ∫10y=0 1/20 dy = 1/2Thus, E(X/Y) = 5.Variance of X/Y is given by:Var(X/Y) = E(X²/Y²) − E(X/Y)²

We can find E(X²/Y²) as follows: Since X and Y are independent, we have: Now,E(X²) = ∫201x=0 x²/20 dx = 200/3

Similarly(Y²) = ∫201y=0 y²/20 dy = 200/3

Thus, E(X²/Y²) = 200/9 And, Var(X/Y) = 200/9 − 5² = 25/9.c) For k > 0 and 1 > 0, compute E((0 − 1)X/Yk).E((0 − 1)X/Yk) = (−1)E(X/Yk) = (−1)E(X)E(1/Yk)Since E(1/Yk) = ∫20y=0 1/20 (y−k)dy = [1/2 − (k/20)ln(1 + 20/k)]

Thus,E((0 − 1)X/Yk) = (−1)(10)[1/2 − (k/20)ln(1 + 20/k)] = 5k ln(1 + 20/k) − 5.d) Find the density function of Z = X/Y.

Since X and Y are independent and uniform [0, 20], the joint pdf of (X, Y) is fXY(x, y) = 1/400 for 0 ≤ x ≤ 20, 0 ≤ y ≤ 20.The region on which the joint density is positive is the square [0, 20] × [0, 20],

so the marginal density functions are: fX(x) = ∫20y=0 1/400 dy = 1/20 for 0 ≤ x ≤ 20fY(y) = ∫20x=0 1/400 dx = 1/20 for 0 ≤ y ≤ 20.We can write the density function of Z as: for 0 ≤ z ≤ 1, and 0 otherwise)

Find an expression for c so that X(n)/c is a lower 100(1 − a)% confidence bound for 0, that is, e satisfies Pr (0 > X(n)/c) = 1 − a.As X1, X2, ... Xn are independent and identically distributed uniform [0, 20] random variables, their maximum M is also uniformly distributed on [0, 20], and its distribution function is given by: P(M ≤ m) = (m/20)n for 0 ≤ m ≤ 20.

To find the lower 100(1 − a)% confidence bound for 0, we need to find c such that P(0 > X(n)/c) = 1 − a, or equivalently, P(X(n)/c > 0) = a. We have: P(X(n)/c > 0) = P(X1/c > 0, X2/c > 0, ..., Xn/c > 0) = P(X1 > 0, X2 > 0, ..., Xn > 0) = (1/20)n

Thus, we need to solve:(1/20)n = a, or equivalently: c = 20(a)−1/n.

To know more about distributed visit:

https://brainly.com/question/29664127

#SPJ11

Find the equation of a line that is perpendicular to the line x = -5 and contains the point (3,-5). The equation of the perpendicular line is __ (Type your answer in standard form, using integer coefficients with A ≥ 0.)

Answers

The equation of the line perpendicular to x = -5 and passing through the point (3, -5) is x = 3.

The line x = -5 is a vertical line parallel to the y-axis, passing through the point (-5, y) for all y-values. A line perpendicular to this line will be a horizontal line parallel to the x-axis.

Since the line passes through the point (3, -5), the x-coordinate remains constant at 3 for all points on the line. Therefore, the equation of the perpendicular line is x = 3. In standard form, this can be written as 1x + 0y = 3, or simply x - 3 = 0.

To learn more about perpendicular click here:

brainly.com/question/12746252

#SPJ11

Question 4 Let (V₁, V₂, V3) be a basis for R3, and let (U₂, U₂, U3) be the orthogonal basis for R constructed by the Gram-Schmidt process. If \
va (1.0.0) and u₁-(1/3,1/3,1/3). U₂-(1/6,1/6,-1/3). (Please use the above format for a fraction and a vector, only a comma between two numbers, no decimals.) (a) The vector U3
(b) Let x be the square of the distance between u1 and U₂, and let cos(θ)=Y. where is the angle between us and us. Then xy 4 poli

Answers

The vector U₃ is (1/6, 1/6, 4/3). To find the vector U₃, we need to apply the Gram-Schmidt process to the given basis vectors.

Let's start with the vector U₁ and U₂

U₁ = (1/3, 1/3, 1/3)

U₂ = (1/6, 1/6, -1/3)

The orthogonal vector U₃ is obtained by subtracting the projection of U₁ onto U₂ from U₁:

U₃ = U₁ - proj(U₁, U₂)

To calculate the projection of U₁ onto U₂, we use the formula:

proj(U₁, U₂) = (U₁ · U₂) / ||U₂||² * U₂

where "·" denotes the dot product and "|| ||" denotes the norm (magnitude) of a vector.

Let's calculate the projection:

U₁ · U₂ = (1/3)(1/6) + (1/3)(1/6) + (1/3)(-1/3) = 1/6 + 1/6 - 1/9 = 1/3

||U₂||² = (1/6)² + (1/6)² + (-1/3)² = 1/36 + 1/36 + 1/9 = 1/9

Now we can calculate the projection:

proj(U₁, U₂) = (1/3) / (1/9) * (1/6, 1/6, -1/3) = 3/1 * (1/6, 1/6, -1/3) = (1/2, 1/2, -1)

Finally, we can calculate U₃:

U₃ = U₁ - proj(U₁, U₂) = (1/3, 1/3, 1/3) - (1/2, 1/2, -1) = (1/6, 1/6, 4/3)

Therefore, the vector U₃ is (1/6, 1/6, 4/3).

(b) To find the square of the distance between U₁ and U₂ (x²) and the cosine of the angle between U₁ and U₃ (cos(θ) = Y), we can use the following formulas:

x² = ||U₁ - U₂||²

cos(θ) = (U₁ · U₃) / (||U₁|| ||U₃||)

Let's calculate them:

||U₁ - U₂||² = ||(1/3, 1/3, 1/3) - (1/6, 1/6, -1/3)||² = ||(1/6, 1/6, 2/3)||² = (1/6)² + (1/6)² + (2/3)² = 1/36 + 1/36 + 4/9 = 9/36 = 1/4

(U₁ · U₃) = (1/3)(1/6) + (1/3)(1/6) + (1/3)(4/3) = 1/18 + 1/18 + 4/9 = 1/6 + 4/9 = 9/54 + 24/54 = 33/54

||U₁|| = ||(1/3, 1/3, 1/3)|| = √((1/3)² + (1/3)² + (1/3)²) = √(1/9 + 1/9 + 1/9) = √(3/9) = √(1/3) = 1/√3

||U₃|| = ||(1/6, 1/6, 4/3)|| = √((1/6)² + (1/6)² + (4/3)²) = √(1/36 + 1/36 + 16/9) = √(18/36) = √(1/2) = 1/√2

Now we can calculate cos(θ):

cos(θ) = (U₁ · U₃) / (||U₁|| ||U₃||) = (33/54) / ((1/√3) * (1/√2)) = (33/54) * (√3/√2) = (11/18) * (√3/√2) = (11√3) / (18√2)

Therefore, the square of the distance between U₁ and U₂ (x²) is 1/4, and cos(θ) (Y) is (11√3) / (18√2).

Learn more about vectors here:

https://brainly.com/question/29740341

#SPJ11

For the following matrix, one of the eigenvalues is repeated. A1 = (-1 -6 2)
(0 2 -1)
(0 -9 2) (a) What is the repeated eigenvalue λ __ and what is the multiplicity of this eigenvalue ___? (b) Enter a basis for the eigenspace associated with the repeated eigenvalue For example, if your basis is {(1, 2, 3), (3, 4, 5)}, you would enter [1,2,3], [3,4,5] (c) What is the dimension of this eigenspace? ___ (d) Is the matrix diagonalisable? a. True b. False

Answers

(a) The repeated eigenvalue is λ = -1, and its multiplicity is 2.

(b) A basis for the eigenspace associated with the repeated eigenvalue is [6, 1, 3].

(c) The dimension of this eigenspace is 1.

(d) False, the matrix is not diagonalizable.

(a) To find the repeated eigenvalue and its multiplicity, we need to calculate the eigenvalues of matrix A. The eigenvalues satisfy the equation |A - λI| = 0, where A is the matrix, λ is the eigenvalue, and I is the identity matrix.

Calculating |A - λI| = 0, we get the characteristic equation:

| (-1-λ) -6   2 |

|   0     2-λ -1 |

|   0     -9   2-λ| = 0

Expanding this determinant and simplifying, we have:

(λ+1)((λ-2)(λ-2) - (-1)(-9)) = 0

(λ+1)(λ² - 4λ + 4 + 9) = 0

(λ+1)(λ² - 4λ + 13) = 0

Solving this equation, we find two roots: λ = -1 and λ = 2. Since the eigenvalue -1 appears twice, it is the repeated eigenvalue with a multiplicity of 2.

(b) To find a basis for the eigenspace associated with the repeated eigenvalue -1, we need to find the null space of the matrix (A - (-1)I), where I is the identity matrix.

(A - (-1)I) = [0 -6 2]

             [0  3 -1]

             [0 -9 3]

Reducing this matrix to row-echelon form, we have:

[0 -6 2]

[0  3 -1]

[0  0  0]

From this, we can see that the third row is a linear combination of the first two rows. Thus, the eigenspace associated with the repeated eigenvalue -1 has dimension 1. A basis for this eigenspace can be obtained by setting a free variable, such as the second entry, to 1 and solving for the remaining variables. Taking the second entry as 1, we obtain [6, 1, 3] as a basis for the eigenspace.

(c) The dimension of the eigenspace associated with the repeated eigenvalue -1 is 1.

(d) False, the matrix A is not diagonalizable because it has a repeated eigenvalue with a multiplicity of 2, but its associated eigenspace has dimension 1.

To learn more about eigenvalue  Click Here: brainly.com/question/32575123

#SPJ11

True or false :- Given the difference quotient, the equation (5(-2 + h)^3 + 40)/ h of the function is y=5x^3

Answers

The statement is false. The given difference quotient, [tex](5(-2 + h)^3 + 40)/h,[/tex]does not simplify to y = [tex]5x^3.[/tex]

To determine whether the given difference quotient simplifies to y = 5x^3, we need to evaluate the expression and compare it with the given equation. Let's simplify the difference quotient:

[tex](5(-2 + h)^3 + 40)/h[/tex]

Expanding the cube, we have:

(5(-8 + 12h - 6h^2 + h^3) + 40)/h

Simplifying further:

[tex](-40 + 60h - 30h^2 + 5h^3 + 40)/h[/tex]

Combining like terms:

[tex](5h^3 - 30h^2 + 60h)/h[/tex]

Now, we can cancel out h from the numerator and denominator:

[tex]5h^2 - 30h + 60[/tex]

The resulting expression, 5h^2 - 30h + 60, does not match the equation y = 5x^3. Therefore, the given difference quotient does not simplify to y = 5x^3. It's important to note that the difference quotient represents the average rate of change of a function, while the equation y = 5x^3 represents a specific function of a single variable.

Learn more about quotient here:

https://brainly.com/question/16134410

#SPJ11

The population of a rare species of flightless birds in 2007 was estimated to be 160,978 birds. By 2014, the number of birds had grown to 218,267. (a) Assuming the population grows linearly, find the linear model, y = mx + b, representing the population x years since 2000. y = Number x + Number (round m and b to 3 decimal places) (b) Using the linear model from part (a), estimate the population in 2030. Number (round to the nearest whole number)

Answers

The linear model representing the population x years since 2000 is y = 13,824.857x + 160,978.000. Using the linear model from part (a), the estimated population in 2030 is 307,602 birds.

(a) To find the linear model, we need to determine the slope (m) and y-intercept (b). We can use the given data points (2007, 160,978) and (2014, 218,267) to calculate the slope:

m = (218,267 - 160,978) / (2014 - 2007) = 13,824.857

Next, we can substitute one of the data points into the equation y = mx + b to solve for the y-intercept:

160,978 = 13,824.857 * 2007 + b

b = 160,978 - (13,824.857 * 2007) = 160,978 - 27,715,715.999 = 160,978.000

Therefore, the linear model representing the population x years since 2000 is y = 13,824.857x + 160,978.000 (rounded to 3 decimal places).

(b) To estimate the population in 2030, we need to substitute x = 2030 - 2000 = 30 into the linear model:

y = 13,824.857 * 30 + 160,978.000 = 414,745.714 + 160,978.000 = 575,723.714

Rounding this to the nearest whole number, the estimated population in 2030 is 575,724 birds.

Learn more about slope here:

https://brainly.com/question/29027297

#SPJ11

Solve the system. (If there are infinitely many solutions, enter INFINITELY MANY. If there is no solution, enter NO SOLUTION.) {4x + 5y = 6 {3x- 2y = 39
(x, y) = ( )

Answers

The system of equations has no solution. There are no values of x and y that satisfy both equations simultaneously.

The system of equations given is:

{4x + 5y = 6

{3x - 2y = 39

To solve this system, we can use the method of substitution or elimination. Let's solve it using the method of elimination:

Multiplying the second equation by 2 gives us:

{6x - 4y = 78

Now, we can subtract the modified second equation from the first equation:

(4x + 5y) - (6x - 4y) = 6 - 78

4x + 5y - 6x + 4y = -72

-2x + 9y = -72

Simplifying further, we get:

-2x + 9y = -72

Now, we have a single equation with two variables. This equation represents a line. However, since we have two variables and only one equation, we can't determine a unique solution. The system is inconsistent, which means there is no solution.

Therefore, the solution to the system of equations is NO SOLUTION

To learn more about system of equations click here :

brainly.com/question/9351049

#SPJ11

pick one of the two companies and sketch out a normal curve for it. be sure to label it and use vertical lines to locate the mean and 1 standard deviation on either side of the mean.

Answers

A normal curve for one of the two companies with labels and vertical lines indicating the mean and 1 standard deviation on either side of the mean.

What is a normal curve A normal curve is a bell-shaped curve with most of the scores clustering around the mean. It is also known as a normal distribution. It has the following characteristicsThis rule states that:Approximately 68% of the data falls within one standard deviation of the mean.Approximately 95% of the data falls within two standard deviations of the mean.Approximately 99.7% of the data falls within three standard deviations of the mean.

Now, coming back to the question. Since the companies are not given, I will choose a random company. Let's assume that the company is ABC Ltd. The mean of the data is 65 and the standard deviation is 5. We have to sketch the normal curve for this data.

To know more about plotting data visit:

https://brainly.com/question/29096917

#SPJ11


Answer the following question regarding the normal
distribution:
Suppose X is a normally distributed random variable with mean 5.
If P(X > 9) = 1/5 calculate the variance of X

Answers

The variance of X is 0.94, given that X is a normally distributed random variable with mean 5, and P(X > 9) = 1/5.

In probability theory and statistics, normal distribution is a continuous probability distribution that describes a symmetric probability distribution whose probability density function (PDF) has a bell-shaped curve with the mean and the standard deviation as its parameters.

The mean represents the center of the distribution, while the standard deviation controls the spread or variance of the distribution.

Suppose X is a normally distributed random variable with mean 5, and P(X > 9) = 1/5, to calculate the variance of X, we must follow these steps:

Step 1: Find the z-score. A z-score is a measure of how many standard deviations above or below the mean a data point is.

Using the standard normal distribution, we can find the z-score corresponding to P(X > 9) = 1/5 as follows:

P(X > 9) = 1/5

P(Z > (9 - 5) / σ) = 1/

P(Z > 1.6 / σ) = 1/5

Using the standard normal distribution table, we can find the corresponding z-score to be 1.645.

Thus,1.645 = {1.6}/{σ}

σ = {1.6}/{1.645} = 0.97

Step 2: Calculate the variance of X.The variance is given by the formula:

{ Var}(X) = σ^2

Substituting the value of σ, we get:

{Var}(X) = 0.97^2 = 0.94

Therefore, the variance of X is 0.94, given that X is a normally distributed random variable with mean 5, and P(X > 9) = 1/5.

Know more about the variance

https://brainly.com/question/9304306

#SPJ11

Find the absolute minima and maxima of the function f(x, y) = x² - 2xy + xy³/2 on the closed region in the xy-plane bounded below by the parabola y = x² and above by the line y = 4. Determine all the points at which the absolute minima and maxima occur.

Answers

To find the absolute minima and maxima of the function f(x, y) = x² - 2xy + xy³/2 on the given region, we need to consider the critical points inside the region and the points on the boundary.

1. Critical Points:

To find the critical points, we need to find the partial derivatives of f(x, y) with respect to x and y and set them equal to zero:

∂f/∂x = 2x - 2y + (3/2)xy² = 0

∂f/∂y = -2x + (3/2)x³ = 0

Solving these equations simultaneously, we get two critical points: (0, 0) and (2/√3, 4/(3√3)).

2. Boundary Points:

We need to evaluate the function f(x, y) at the points on the boundary of the given region.

a) Along the parabola y = x²:

Substituting y = x² into f(x, y), we get f(x) = x² - 2x³ + (x⁵/2). To find the absolute extrema on the parabola, we need to find the critical points of f(x).

Taking the derivative of f(x) with respect to x and setting it equal to zero:

f'(x) = 2x - 6x² + (5x⁴/2) = 0

Solving this equation, we get the critical points: x = 0, x = 2/√5, x = -2/√5.

b) Along the line y = 4:

Substituting y = 4 into f(x, y), we get f(x) = x² - 8x + 8. To find the absolute extrema on the line, we need to find the critical points of f(x).

Taking the derivative of f(x) with respect to x and setting it equal to zero:

f'(x) = 2x - 8 = 0

Solving this equation, we get the critical point: x = 4.

Determining Absolute Extrema:

Now we compare the values of f(x, y) at the critical points and the boundary points to determine the absolute extrema.

The critical points are:

(0, 0): f(0, 0) = 0

(2/√3, 4/(3√3)): f(2/√3, 4/(3√3)) ≈ -0.154

On the parabola y = x²:

x = 0: f(0) = 0

x = 2/√5: f(2/√5) ≈ -1.867

x = -2/√5: f(-2/√5) ≈ -1.867

On the line y = 4:

x = 4: f(4) = -8

Comparing these values, we find that the absolute minimum is approximately -8 at the point (4, 4) on the line y = 4. There are no absolute maximum values within the given region.

Therefore, the absolute minimum occurs at the point (4, 4) on the line y = 4.

To know more about absolute visit-

brainly.com/question/31502587

#SPJ11

Use the points (-3, 4) and (4, -2) to answer parts a)-e). (5 points each)
a) Graph the line that passes through the two points. Be sure to label the scale and both axes.
b) Find the slope.

Answers

a) To graph the line that passes through the points (-3, 4) and (4, -2), we can plot these points on a coordinate plane and then draw a straight line that connects them.

Using the given points, we plot (-3, 4) and (4, -2) on the coordinate plane. We label the x-axis and y-axis with appropriate scales to ensure accuracy. Then, we draw a straight line passing through these two points. The resulting graph represents the line that passes through the given points.

b) To find the slope of the line passing through the points (-3, 4) and (4, -2), we can use the slope formula:

slope = (change in y)/(change in x) = (y₂ - y₁)/(x₂ - x₁).

Substituting the coordinates of the given points, we have:

slope = (-2 - 4)/(4 - (-3)) = (-2 - 4)/(4 + 3) = (-6)/(7).

Hence, the slope of the line passing through the points (-3, 4) and (4, -2) is -6/7.

To graph the line passing through the given points, we plot (-3, 4) and (4, -2) on a coordinate plane and connect them with a straight line. The slope of the line is -6/7, which represents the ratio of the vertical change (change in y) to the horizontal change (change in x) between the two points.

To learn more about coordinate plane click here:

brainly.com/question/14462788

#SPJ11

A square is inscribed in a circle. if the area of the square is 9in^2
, phi r^2 what is the ratio of the circumference of the circle to the area of the circle?

Answers

Therefore, the ratio of the circumference of the circle to the area of the circle is (2/3)√2.

To find the ratio of the circumference of the circle to the area of the circle, we need to determine the properties of the circle.

Let's assume that the side length of the square inscribed in the circle is 's'. Since the area of the square is given as 9 square inches, we have s^2 = 9.

Making the square root of both sides, we find that s = 3.

The diagonal of the square is equal to the diameter of the circle, which can be found using the Pythagorean theorem. The diagonal is given by d = s√2 = 3√2.

The radius of the circle is half the diameter, so the radius is r = (1/2) * 3√2 = (3/2)√2.

The circumference of the circle is given by C = 2πr = 2π * (3/2)√2 = 3π√2.

The area of the circle is given by A = πr^2 = π * ((3/2)√2)^2 = 9/2 * π.

Now, we can calculate the ratio of the circumference to the area:

C/A = (3π√2) / (9/2 * π)

= (6/9)√2

= (2/3)√2.

To know more about circumference of the circle,

https://brainly.com/question/24033860

#SPJ11


Use the Gauss-Seidel iterative technique to find approximate
solutions to the following:
2x1 + x2 − 2x3 = 1
2x1 − 3x2 + x3 = 0
x1 − x2 + 2x3 = 2
with X = (0, 0, 0, 0)

Answers

The Gauss-Seidel iterative technique is a method used to solve a system of linear equations. Here’s the approximate solutions (0.5, 0.333, 0.917).

To begin, reorganise the equations in such a way that the element that represents the diagonal is on the left side, and move every other element to the right side: x1 = (1 - x2 + 2x3)/2 x2 = (2x1 + x3)/3 x3 = (2 - x1 + x2)/2

The next thing that needs to be done is to take the value that has been provided, which is (0, 0, 0), as an initial guess for the solution vector x. Iterate using the equations from the previous step until you reach a point of convergence, and then go to the next step. The example that follows provides an illustration of what the first version of the product would look like:

x1 = (1 - 0 + 20)/2 = 0.5 x2 = (20.5 + 0)/3 = 0.333 x3 = (2 - 0.5 + 0.333)/2 = 0.917

After the conclusion of one cycle, the values (0.5, 0.333, 0.917) are assigned to the solution vector x. This change takes effect immediately. You are at liberty to continue iterating until you have achieved the level of precision that is necessary for your purposes.

Learn more about Gauss-Seidel iterative here:

https://brainly.com/question/31310894

#SPJ11

Find and classify the critical points of f(x, y) = - 4xy - x³ - 2y². For each type of behavior, enter a list of ordered pairs where the If there are no points where the behavior occurs, enter "DNE" f(x, y) has a local maximum at_____
f(x, y) has a local minimum at______
f(x, y) has a saddle point at________

Answers

In summary: f(x, y) has a local maximum at DNE (since there are no points of local maximum).  f(x, y) has a local minimum at DNE (since there are no points of local minimum). f(x, y) has a saddle point at (0, 0).

To find and classify the critical points of the function f(x, y) = -4xy - x³ - 2y², we need to find the points where the gradient of the function is zero or undefined.

Taking the partial derivatives with respect to x and y:

∂f/∂x = -4y - 3x²

∂f/∂y = -4x - 4y

Setting both partial derivatives to zero, we have:

-4y - 3x² = 0 ...(1)

-4x - 4y = 0 ...(2)

Solving equations (1) and (2) simultaneously, we get:

x = 0

y = 0

So, the critical point is (0, 0).

To classify the critical point, we need to determine the nature of the critical point by examining the second-order partial derivatives.

Taking the second partial derivatives:

∂²f/∂x² = -6x

∂²f/∂y² = -4

∂²f/∂x∂y = -4

Evaluating the second partial derivatives at the critical point (0, 0), we have:

∂²f/∂x² = 0

∂²f/∂y² = -4

∂²f/∂x∂y = -4

Using the second partial derivative test, we can classify the critical point:

If ∂²f/∂x² > 0 and (∂²f/∂x²)(∂²f/∂y²) - (∂²f/∂x∂y)² > 0, it is a local minimum.

If ∂²f/∂x² < 0 and (∂²f/∂x²)(∂²f/∂y²) - (∂²f/∂x∂y)² > 0, it is a local maximum.

If (∂²f/∂x²)(∂²f/∂y²) - (∂²f/∂x∂y)² < 0, it is a saddle point.

At the critical point (0, 0), we have:

∂²f/∂x² = 0

∂²f/∂y² = -4

∂²f/∂x∂y = -4

Since ∂²f/∂x² = 0 and (∂²f/∂x²)(∂²f/∂y²) - (∂²f/∂x∂y)² = 0 - (-4)(-4) = -16 < 0, the critical point (0, 0) is a saddle point.

To know more about saddle point,

https://brainly.com/question/31399853

#SPJ11

A = [-2 2]
[-1 3]
B = [2 4]
[3 1]
[1 1]
For the matrices A and B given, find BA if possible. a. [-4 8]
[-3 3]
[ 1 1] b. [-6 14]
[-7 12]
[-3 5]
c. [-8 16]
[-7 9]
[-3 5]
d. Not possible.

Answers

The product of matrices B and A, denoted as BA, is not possible. Therefore, the correct answer is option d: Not possible. To multiply two matrices, their dimensions must be compatible.

1. For matrix B with dimensions 3x2 and matrix A with dimensions 2x2, the number of columns in matrix B must match the number of rows in matrix A for the multiplication to be valid.

2. In this case, matrix B has 2 columns, and matrix A has 2 rows, which satisfies the condition for matrix multiplication. However, the product of B and A would result in a matrix with dimensions 3x2, which does not match the dimensions of matrix B.

3. Hence, BA is not possible, and the answer is option d: Not possible.

learn more about matrix multiplication here: brainly.com/question/13591897

#SPJ11

The random variable X has range (0, 1), and p.d.f. given by f(x)
= 12x2 (1 − x), 0
The mean of X is equal to 3/5 .calculate E(X^2) and hence
V(X)

Answers

The value of E(x²) = 2/5 and the value of V(X) = 1/25, for the random variable X.

To calculate E(X²), we need to find the expected value of X². We can use the formula:

E(X²) = ∫[x² * f(x)] dx

Given that the probability density function (PDF) is:

f(x) = 12x²(1 - x), 0 < x < 1

We can calculate E(X²) as follows:

E(X²) = ∫[x² * 12x²(1 - x)] dx

= 12∫[x⁴ - x⁵] dx

= 12[(1/5)x⁵ - (1/6)x⁶] evaluated from 0 to 1

= 12[(1/5)(1⁵) - (1/6)(1⁶)] - 12[(1/5)(0⁵) - (1/6)(0⁶)]

= 12[(1/5) - (1/6)] - 12[0 - 0]

= 12[(6 - 5)/30]

= 12/30

= 2/5

Therefore, E(X²) is equal to 2/5.

To calculate V(X) (the variance of X), we can use the formula:

V(X) = E(X²) - [E(X)]²

Given that the mean of X is 3/5, we can substitute the values:

V(X) = 2/5 - [(3/5)²]

= 2/5 - 9/25

= 10/25 - 9/25

= 1/25

Therefore, V(X) is equal to 1/25.

To learn more about Probability Density Function(p.d.f): https://brainly.com/question/30403935

#SPJ11




Find IAI, IBI, AB, and IABI. Then verify that IA||B| = |AB|. 4 0 1 1 1 0 -1 1 1 -1 0 1 4 1 0 4 A = 4 2 1 0 1 1 1 0 1 4 20 2 4 10 (a) |A| (b) |B| (c) AB 0000 (d) |AB| 00 || 0000

Answers

To find the values of |A|, |B|, AB, and |AB|, we perform the following calculations:

(a) |A|: The determinant of matrix A

|A| = 4(1(4) - 1(1)) - 2(1(4) - 1(1)) + 1(1(1) - 4(1))

= 4(3) - 2(3) + 1(-3)

= 12 - 6 - 3

= 3

Therefore, |A| = 3.

(b) |B|: The determinant of matrix B

|B| = 0(1(4) - 1(1)) - 1(1(4) - 1(1)) + 1(1(1) - 4(0))

= 0(3) - 1(3) + 1(1)

= 0 - 3 + 1

= -2

Therefore, |B| = -2.

(c) AB: The matrix product of A and B

AB = (4(4) + 0(1) + 1(1)) (4(0) + 0(1) + 1(1)) (4(1) + 0(1) + 1(1))

= (16 + 0 + 1) (0 + 0 + 1) (4 + 0 + 1)

= 17 1 5

Therefore, AB =

| 17 1 5 |.

(d) |AB|: The determinant of matrix AB

|AB| = 17(1(5) - 1(1)) - 1(1(5) - 1(1)) + 5(1(1) - 5(0))

= 17(4) - 1(4) + 5(1)

= 68 - 4 + 5

= 69

Therefore, |AB| = 69.

Now, let's verify that |A|||B| = |AB|:

|A|||B| = 3|-2|

= 3(2)

= 6

|AB| = 69

Since |A|||B| = |AB|, the verification is correct.

To summarize:

(a) |A| = 3

(b) |B| = -2

(c) AB =

| 17 1 5 |

(d) |AB| = 69

The calculations and verifications are complete.

To know more about Matrix visit-

brainly.com/question/28180105

#SPJ11

Everyone is familiar with waiting lines or queues. For example, people wait in line at a supermarket to go through the checkout counter. There are two factors that determine how long the queue becomes. One is the speed of service. The other is the number of arrivals at the checkout counter. The mean number of arrivals is an important summary statistic, but so is the standard deviation. A consultant working for the supermarket counted the number of arrivals (shown below) per hour during a sample of 30 hours. 109 105 106 97 103 132 91 89 99 115 111 106 84 101 75 102 94 130 84 72 71 88 107 95 98 93 101 98 94 90 Assuming data is normally distributed (i.e. histogram is bell shaped) and given the mean and standard deviation calculated, usually what range of number of arrivals do you expect for this supermarket? (Remember "usually" means 95% of the time). OA 84 to 112 B. 70 to 126 c. 56 to 140 0.71 to 132 E. 70 to 162

Answers

The range of number of arrivals you can expect for this supermarket, usually 95% of the time, is 70 to 126.

To determine the range of number of arrivals expected at the supermarket, given the mean and standard deviation, we can use the concept of the normal distribution. Assuming the data is normally distributed, we can calculate the range that includes 95% of the data, which is the usual range. The answer options provided represent different ranges of number of arrivals. We need to identify the range that falls within the 95% confidence interval of the data.

To find the range of number of arrivals expected with 95% confidence, we can use the mean and standard deviation of the sample. The mean represents the average number of arrivals, and the standard deviation measures the dispersion of the data.

Since the data is assumed to follow a normal distribution, we know that approximately 95% of the data falls within two standard deviations of the mean. This means that the expected range will be the mean plus or minus two standard deviations.

To calculate this range, we can add and subtract two times the standard deviation from the mean. Using the given mean and standard deviation, we can determine the lower and upper limits of the expected range.

Comparing the answer options provided, we need to choose the range that falls within the calculated range. The option that matches the calculated range would be the correct answer, representing the range of number of arrivals we expect at the supermarket with 95% confidence.

Learn more about confidence interval here:

https://brainly.com/question/32546207

#SPJ11

In the previous question, write your answer in the standard form (namely, enter your answer exactly in the form of Ax + By = C) and also simplify as much as possible. The enter your equation below. Do not type any spaces or extra character. Find the equation of a line passing through (3,4) and (1,-4). Enter your answer in the slope-intercept form (namely, type your answer exactly in the form of y=mx+b).

Answers

It should be noted that the equation of the line passing through the points (3, 4) and (1, -4) is y = 4x - 8.

How to explain the equation

In order to find the equation of a line passing through two points, (x₁, y₁) and (x₂, y₂), you can use the point-slope form of the equation, which is:

y - y₁ = m(x - x₁),

where m is the slope of the line.

Given the points (3, 4) and (1, -4), we can calculate the slope (m) using the formula:

m = (y₂ - y₁) / (x₂ - x₁).

Plugging in the values:

m = (-4 - 4) / (1 - 3) = -8 / -2

= 4.

Now that we have the slope (m) and one of the points (3, 4), we can use the point-slope form to write the equation of the line:

y - 4 = 4(x - 3).

Simplifying:

y - 4 = 4x - 12.

Moving the constant term to the right side:

y = 4x - 12 + 4.

y = 4x - 8.

Therefore, the equation of the line passing through the points (3, 4) and (1, -4) is y = 4x - 8.

Learn more about equations on

https://brainly.com/question/2972832

#SPJ4

If f(x) = 2x² 2x² - 4x + 4, find ƒ'( – 5). = _____
Use this to find the equation of the tangent line to the parabola y 2x² - 4x + 4 at the point ( – 5, 74). The equation of this tangent line can be written in the form y = mx + b where m is: and where b is:

Answers

In this equation, the value of m (slope) is -24, and the value of b (y-intercept) is 46.

To find ƒ'(–5), we need to find the derivative of the function f(x) = 2x² - 4x + 4 and evaluate it at x = -5.

Let's find the derivative of f(x) step by step:

f(x) = 2x² - 4x + 4

Using the power rule, the derivative of x^n with respect to x is nx^(n-1), where n is a constant:

f'(x) = d/dx (2x²) - d/dx (4x) + d/dx (4)

f'(x) = 4x^1 - 4 + 0

f'(x) = 4x - 4

Now, let's evaluate f'(x) at x = -5:

f'(-5) = 4(-5) - 4

f'(-5) = -20 - 4

f'(-5) = -24

So, ƒ'(-5) = -24.

To find the equation of the tangent line to the parabola at the point (-5, 74), we have the point (-5, 74) and the slope of the tangent line, which is m = ƒ'(-5) = -24.

Using the point-slope form of the equation of a line:

y - y₁ = m(x - x₁)

where (x₁, y₁) is the given point and m is the slope, we can substitute the values:

y - 74 = -24(x - (-5))

y - 74 = -24(x + 5)

y - 74 = -24x - 120

Rearranging the equation to the slope-intercept form (y = mx + b):

y = -24x + 46

the equation of the tangent line to the parabola y = 2x² - 4x + 4 at the point (-5, 74) is y = -24x + 46.

To know more about derivative visit:

brainly.com/question/29144258

#SPJ11

The PTO is selling raffle tickets to raise money for classroom supplies. A raffle ticket costs $4. There is 1 winning ticket out of the 110 tickets sold. The winner gets a prize worth $82. Round your answers to the nearest cent.

What is the expected value (to you) of one raffle ticket? $

Calculate the expected value (to you) if you purchase 10 raffle tickets. $

What is the expected value (to the PTO) of one raffle ticket? $

If the PTO sells all 110 raffle tickets, how much money can they expect to raise for the classroom supplies? $

Answers

The z-score for P(? ≤ z ≤ ?) = 0.60 is approximately 0.25.

The z-score for P(z ≥ ?) = 0.30 is approximately -0.52.

How to find the Z score

P(Z ≤ z) = 0.60

We can use a standard normal distribution table or a calculator to find that the z-score corresponding to a cumulative probability of 0.60 is approximately 0.25.

Therefore, the z-score for P(? ≤ z ≤ ?) = 0.60 is approximately 0.25.

For the second question:

We want to find the z-score such that the area under the standard normal distribution curve to the right of z is 0.30. In other words:

P(Z ≥ z) = 0.30

Using a standard normal distribution table or calculator, we can find that the z-score corresponding to a cumulative probability of 0.30 is approximately -0.52 (since we want the area to the right of z, we take the negative of the z-score).

Therefore, the z-score for P(z ≥ ?) = 0.30 is approximately -0.52.

Read more on Z score here: brainly.com/question/25638875

#SPJ1

Sequences and series- Grade 11 math please answer as detailed and clear as possible! 9. Liam is the foreman for a new lake being excavated. One day 1.6 ton of material is removed from the lake bed. Each day following 5%o more is removed than the previous day. What is the amount removed on the 30tday?Show and EXPLAIN all steps to getfullmarks

Answers

To find the amount of material removed on the 30th day, we can use the concept of a geometric sequence.

In this scenario, each day the amount removed increases by 5%o (which means 5% of the previous day's amount is added). Let's break down the solution into two parts: finding the common ratio and calculating the amount removed on the 30th day.

First, we need to determine the common ratio of the sequence. Since each day 5%o more material is removed than the previous day, the common ratio can be calculated as follows:

Common ratio = 1 + (5%o) = 1 + 0.05 = 1.05

Now, we can use this common ratio to find the amount removed on the 30th day. We know that 1.6 tons of material was removed on the first day. To find the amount removed on the 30th day, we multiply the initial amount by the common ratio raised to the power of (30 - 1) since we want to find the amount after 29 additional days:

Amount on 30th day = 1.6 tons * (1.05)^(30 - 1)

Calculating this expression will give us the amount of material removed on the 30th day.

To know more about Sequences and series click here: brainly.com/question/32549533

#SPJ11

If the measure of arc MOP = 11x-38 and the measure of angle
LMP = 3x+41, find the measure of angle NMP.

Answers

Answer:

mop=11x-38

Lmp=3x+41

we kow that,

area of circle=2pier²

Which of the following would be an appropriate alternative
hypothesis?
The mean of a population is equal to 125.
The mean of a sample is equal to 125.
The mean of a population is gre

Answers

The appropriate alternative hypothesis is "The mean of a population is greater than 125".

An alternative hypothesis is a statement that is formulated to compete with a null hypothesis, and it generally contradicts or negates the null hypothesis.Therefore, the appropriate alternative hypothesis out of the given options would be "The mean of a population is greater than 125".Option A states that the mean of a population is equal to 125, which is similar to the null hypothesis, so it cannot be an alternative hypothesis.Option B states that the mean of a sample is equal to 125, which cannot be considered an appropriate alternative hypothesis as it is about a sample, not a population.The last option C is also incomplete, and thus, it cannot be considered as an alternative hypothesis.

An alternative hypothesis is a statement that is formulated to compete with a null hypothesis, and it generally contradicts or negates the null hypothesis.Therefore, the appropriate alternative hypothesis out of the given options would be "The mean of a population is greater than 125".Option A states that the mean of a population is equal to 125, which is similar to the null hypothesis, so it cannot be an alternative hypothesis.Option B states that the mean of a sample is equal to 125, which cannot be considered an appropriate alternative hypothesis as it is about a sample, not a population.The last option C is also incomplete, and thus, it cannot be considered as an alternative hypothesis.

To know more about alternative hypothesis visit :-

https://brainly.com/question/30404845

#SPJ11

Find an equation of the ellipse having a major axis of length 10 and foci at (9, 2) and (1, 2).

Answers

The equation of the ellipse with a major axis length of 10 and foci at (9, 2) and (1, 2) is ((x - 5)^2)/25 + ((y - 2)^2)/9 = 1.

To find the equation of the ellipse, we need to determine its center, major and minor axes lengths, and the orientation. Since the foci lie on a horizontal line with a common y-coordinate of 2, we can deduce that the major axis is horizontal.

The distance between the foci is 9 units, which is equal to the length of the major axis. Therefore, the distance from the center to each focus is half the length of the major axis, i.e., 9/2 = 4.5 units. The center of the ellipse lies midway between the foci, so its x-coordinate is the average of the x-coordinates of the foci, which is (9 + 1)/2 = 5. The y-coordinate of the center is the same as that of the foci, which is 2.

We can now write the equation of the ellipse using the formula:

((x - h)^2)/a^2 + ((y - k)^2)/b^2 = 1,

where (h, k) represents the center of the ellipse, and a and b are the semi-major and semi-minor axes, respectively.

Plugging in the values, we get:

((x - 5)^2)/a^2 + ((y - 2)^2)/b^2 = 1.

To determine the values of a and b, we use the fact that the length of the major axis is 10 units. Since a is the semi-major axis, a = 10/2 = 5.

To find the value of b, we use the relationship between the semi-major axis and the distance between the center and each focus. Using the Pythagorean theorem, we can find b as follows:

b^2 = a^2 - c^2,

where c is the distance between the center and each focus. In this case, c = 4.5. Substituting the values, we have:

b^2 = 5^2 - 4.5^2 = 25 - 20.25 = 4.75.

Thus, the equation of the ellipse is ((x - 5)^2)/25 + ((y - 2)^2)/4.75 = 1.

Learn more about equation  here: brainly.com/question/29657983

#SPJ11

Find the constants a and b such that the function is continuous on the entire real line.

g(x) =
e-2x − 6 , x ≤ 0
ax + b , 0 < x < 3
e3-x + 1 , x ≥ 3

Answers

To find the constants a and b such that the function g(x) is continuous on the entire real line, we need to ensure that the function is continuous at the points where the piecewise definition changes.

Continuity at x = 0:

The left-hand limit as x approaches 0 from the negative side should be equal to the value of the function at x = 0.

lim(x→0-) g(x) = lim(x→0-) (e^(-2x) - 6) = e^0 - 6 = 1 - 6 = -5

Therefore, we need to have the following equation: g(0) = a(0) + b = -5

Simplifying this equation, we find: b = -5

Continuity at x = 3:

The left-hand limit as x approaches 3 from the negative side should be equal to the right-hand limit as x approaches 3 from the positive side.

lim(x→3-) g(x) = lim(x→3-) (ax + b) = 3a - 5

The right-hand limit as x approaches 3 from the positive side should be equal to the value of the function at x = 3.

lim(x→3+) g(x) = lim(x→3+) (e^(3-x) + 1) = e^0 + 1 = 1 + 1 = 2

Therefore, we need to have the following equation: 3a - 5 = 2

Simplifying this equation, we find:

3a = 7

a = 7/3

So the constants a and b that make the function g(x) continuous on the entire real line are a = 7/3 and b = -5.

To know more about function is continuous visit:

https://brainly.com/question/30089268

#SPJ11

what is the output of this program?
numa = 10
for count in range(3, 6):
numa = numa count
print(numa)

Answers

The given program utilizes a for loop to perform a specific set of operations. The output of the program will be 600.

A for loop is a control structure in programming that allows repeated execution of a block of code. It typically consists of three components: initialization, condition, and increment/decrement. In this program, the initialization sets 'numa' to 10. The condition specifies the range of values from 3 to 5 using the range() function. The increment is implicit and is defined by the range() function itself.

Within the loop, the statement 'numa = numa * count' updates the value of 'numa' by multiplying it with the current value of 'count'. This operation is performed three times since the loop iterates three times for values 3, 4, and 5. After the loop completes, the final value of 'numa' is printed as the output.

In the first iteration, 'numa' is multiplied by 3: 10 * 3 = 30.

In the second iteration, 'numa' is multiplied by 4: 30 * 4 = 120.

In the third iteration, 'numa' is multiplied by 5: 120 * 5 = 600.

The output of the program will be 600.

Learn more about range here:

https://brainly.com/question/29204101

#SPJ11

Write the correct answer. Use numerals instead of words. If necessary, use / for the fraction bar(s).

(3a³-56³) + ______ a³ + b³ = (2³ + b³).

Answers

The correct answer is:

(3a³ - 56³) + 56³ = 2³ + b³

In this equation, the term (3a³ - 56³) cancels out with the corresponding term 56³ on both sides. This simplifies the equation to:

0 = 2³ + b³

Therefore, the correct answer is:

0 = 8 + b³

Know more about equation here:

https://brainly.com/question/29657983

#SPJ11

prove the polynomial identity. (a−1)3 (a−1)2=a(a−1)2(a−1)3 (a−1)2=a(a−1)2 drag and drop the expressions to correctly complete the proof of the polynomial identity.

Answers

To prove the polynomial identity (a−1)³(a−1)² = a(a−1)²(a−1)³, we can expand both sides of the equation and simplify them to show that they are equal.

Expanding the left side of the equation, we have:

(a−1)³(a−1)² = (a−1)(a−1)(a−1)(a−1)²

Expanding the right side of the equation, we have:

a(a−1)²(a−1)³ = a(a−1)(a−1)(a−1)(a−1)²

Now, let's simplify both sides of the equation:

Left side:

(a−1)(a−1)(a−1)(a−1)² = (a−1)⁴(a−1)² = (a−1)⁶

Right side:

a(a−1)(a−1)(a−1)(a−1)² = a(a−1)³(a−1)² = a(a−1)⁶

Since (a−1)⁶ is common to both sides of the equation, we can conclude that (a−1)³(a−1)² = a(a−1)²(a−1)³ is indeed a valid polynomial identity.

Therefore, by expanding and simplifying both sides of the equation, we have shown that the given polynomial identity holds true.

Learn more about polynomial identity here:

https://brainly.com/question/11491967

#SPJ11

Let L be the line given by the span of [2]
[1]
[9]
1 in R³. Find a basis for the orthogonal complement L⊥ of L. A basis for L⊥ is {[___],[___]}

Answers

In this problem, we are given a line L in R³ spanned by the vector [2][1][9]1. We are asked to find a basis for the orthogonal complement L⊥ of L.

To find the orthogonal complement L⊥, we need to determine the vectors that are orthogonal to every vector in L. The vectors in L⊥ are perpendicular to L and span a subspace that is perpendicular to L.

To find a basis for L⊥, we can use the fact that the dot product of any vector in L⊥ with any vector in L is zero. Let's call the vectors in L⊥ [x][y][z]1.

Taking the dot product of [x][y][z]1 with [2][1][9]1, we get:

2x + y + 9z = 0.

This equation represents a plane in R³. We can choose any two linearly independent vectors in this plane to form a basis for L⊥.

One possible basis for L⊥ is {[1][-2][0]1, [9][-18][2]1}. These two vectors are linearly independent and satisfy the equation 2x + y + 9z = 0. Therefore, they span L⊥, the orthogonal complement of L.

To learn more about orthogonal complement, click here:

brainly.com/question/32196772

#SPJ11

Other Questions
vanhoe Co. estimates that variable costs will be 70% of sales and fixed costs will total $3,040,800. The selling price of the product is $14.00, and 760,000 units will be sold. Using the mathematical equation, (a) Compute the break-even sales units and sales dollars. A 8-year annuity of 16 $8,700 semiannual payments will begin 10 years from now, with the first payment coming 10.5 years from now.If the discount rate is 11 percent compounded semiannually, what is the value of this annuity eight years and six years from now? Do not round intermediate calculations and round your answers to 2 decimal places, e.g., 32.16.What is the value of the annuity today? Do not round intermediate calculations and round your answer to 2 decimal places, e.g., 32.16 As a financial manager, you are attempting to assess the future dividend policy of Moonlight Corp. A close examination of the company's life cycle, you came up with the following preliminary analysis: The company anticipates no payout of earnings in the form of cash dividends during the development stage (I). During the growth stage (II), an anticipation of 10 percent of earnings will be distributed as dividends. As the firm progresses to the expansion stage (III), the payout ratio will go up to 20 percent, and eventually reach 40 percent during the maturity stage (IV). a. Assuming earnings per share will be as follows during each of the four stages, indicate the cash dividend per share (if any) during each stage. 3 marks, b1 Stage I.......... $ 0.50Stage II......... ...2.00Stage III............3.20 Stage IV ...........3.50 b. Assume in Stage IV that an investor owns 200 shares and is in a 20 percent tax bracket; what will be the investor's after-tax income from the cash dividend? 2 marks, b1 c. Analyse the stages when Moonlight Corp most likely to utilize stock dividends or stock splits. Provide a reasonable rationale. 5 marks, c1 Which of the following regions were not visited by Phoenician traders O Babylon O Ireland O SenegalO Sicily The Phoenician script differs from other ancient scripts like Linear-A, Cuneiform and Hieroglyphs in that O it is alphabetic, meaning that each character represents a letter rather than a syllable or word. O it is pictorial, meaning that each character represents a concept O it is cursive, meaning that the letters are joined together. O it is syllabic, meaning that each character denotes a syllable Theatres has determined that the price elasticity of demand for two customer segments (A Student Ticket to a Movie and a General Audience Ticket) is -1.25 and -1.4633. Based on their expectations of profitability, Theatres realizes the price of a General Ticket should be $12.00. How much should Theatres charge for its Student Ticket?MR^A = P_A(1 + 1/_A) = m = P_B(1 + 1/_B) = MR^B Select all the possible components of clouds listed below. (They would not all occur in the same clouds at the same time.) Sales for Triad Inc. have grown from $3.500 million to $9.500 million in 8 years. What is the implied annual growth rate of sales for Triad? a 13.29% b 14.13% c 9.44% d 6.97% Select a local business that you're familiar with (such as your favorite restaurant. your friend's lawn care business, etc). What are its main operations? Pick two of its operations that you think needs improving. How can you improve the selected two operations? What resources are required to improve the operations? Company XYZ made no adjusting entry for accrued and unpaid employee salaries of $5,000 on December 31. The entry to record the adjusting entry should have been: O Debit Salary Expense, $5,000, credit Salaries Payable, $5,000 O Debit Salary Expense, $5,000, credit Cash, $5,000 O Debit Salary Expense, $5,000, credit Fees Eamed, $5,000 Debit Salary Expense, $5,000; credit Prepaid Salary, $5,000 Compare the flowing merchandise purchases and sales during the Apr, 2001 The beginning inventory balance 400 units at $50 each Sold 250 unids at 5.40 On March 1, fixtures and equipment were purchased for $4,000 with a downpayment of $1,500 and a $2,500 note, payable in one year. Interest of 6% per year was due when the note was repaid. The estimated life of the fixtures and equipment is 10 years with no expected salvage value. [Note: Record the complete March 1 entry for the equipment purchase first, the complete March 31 depreciation adjusting entry second, and the complete March 31 interest adjusting entry third.]ACCOUNT: Cash Accounts Receivable Inventory Prepaid Rent Fixtures and Equipment Accounts Payable Interest Payable Wages Payable Notes Payable Paid-in Capital Retained Earnings Leave BlankAccount: Dollar amount:Account: Dollar amount:Account: Dollar amount:Account: Dollar amount:Account: Dollar amount:Account: Dollar amount:Account: Dollar amount:Account: Dollar amount: View Policies Current Attempt in Progress On June 1, Splish Brothers Inc. issues 2,800 shares of no-par common stock at a cash price of $8 per share. Journalize the issuance of the shares. (Credit account titles are automatically indented when amount is entered. Do not indent manually. If no entry is required, select "No Entry" for the account titles and enter for the amounts.) Date Account Titles and Explanation Debit Credit June 1 e Textbook and Media List of Accounts Using the thermodynamic information in the ALEKS Data tab, calculate the standard reaction entropy of the following chemical reaction:2Al(s) + FeO(s) AlO(s) + 2Fe(s)Round your answer to zero decimal places.Ans: ______J/K find the coordinates of the midpoint of pq with endpoints p(5, 1) and q(7, 3). how does a catalyst change the rate of reaction without affecting its equilibrium Ryan acquired a rental property in Noosa under a contract ofpurchase on 20 October 1991 for $325,000. Ryan borrowed $300,000from Westpac to fund the acquisition of the property.Ryan incurred the fo A-2+Which graph represents thefunction y = tan x?B2T2TD-2+1214+2T2T According to a survey, high school girls average 100 text messages daily (The Boston Globe, April 21, 2010). Assume the population standard deviation is 20 text messages. Suppose a random sample of 50 high school girls is taken. [You may find it useful to reference the z table. a. What is the probability that the sample mean is more than 105? (Round "z" value to 2 decimal places, and final answer to 4 decimal places.) Probability b. what is the probability that the sample mean is less than 95? (Round "z" value to 2 decimal places, and final answer to 4 decimal places.) Probability 0.0384 c. What is the probability that the sample mean is between 95 and 105? (Round "z" value to 2 decimal places, and final answer to 4 decimal places.) Probability 0.9232 When a solution is diluted by adding additional solvent, the concentration of solute changes but the amount of solute present does not change. Explain. Applying Normalization to Database Design Provide an example of an entity that violates 1st normal form. Describe the problem and what can be done to correct it. Provide an example of an entity that violates 2nd normal form. Describe the problem and what can be done to correct it. Provide an example of an entity that violates 3rd normal form. Describe the problem and what can be done to correct it What are the two main worries of foreign direct investment? Not yet answered Marked out of 1.00 O a. who receives the profits and who controls the assets and. O b. who finances the loan and who pays the interests. O c. who repatriates the money and who pays the taxes